Search found 12 matches


Hello. Can some one please explain how to solve this question. Store S sold a total of 90 copies of a certain book during the seven days of last week, and it sold different numbers of copies on any two of the days. If for the seven days Store S sold the greatest number of copies on Saturday and the...

by vinamra

Mon Jan 04, 2010 9:01 pm
Forum: Data Sufficiency
Topic: DS question
Replies: 8
Views: 3906

Q6. Cannot Say....... (No concrete Cost figure given) Q16 = 20 days According to question, GBP and Euro continues to rise by same amount ie, 0.42 and 1.68 every day So, lets PLUG-IN 30 days....lets see what happens after 30 days GBP= 70.72+ (0.42 x 30)= 83.32 Euro = 46.61+(1.68 x 30)= 97.01 But, Plu...

by vinamra

Mon Jan 04, 2010 8:58 am
Forum: Problem Solving
Topic: Math graphical questionings
Replies: 1
Views: 14316

Hi, I too just started with GMAT! I got: 1) Cracking the GMAT 2010 by Princeton Review (For quick brush-up of basics and new test taking strategies) 2) Manhattan GMAT prep books, 4th edition 3) OG 12th Edition. I think those are enough material for practice.Also, you get: 1) 2 GMAC's GMAT Prep exams...

by vinamra

Mon Jan 04, 2010 8:01 am
Forum: GMAT Strategy
Topic: Suggest some books or materials for the starter
Replies: 2
Views: 1030

I don't think so! My answer is Option B Explanation : Statement 1 results in various Sales Value on Friday! So option B C E remains Statement 2: 90-Saturday (38)= 52 No Days are equal, so lets plug-in Maximum values for Sun, Mon, Tue, Wen,Thur so that we can get minimum possible of Friday... also re...

by vinamra

Mon Jan 04, 2010 5:07 am
Forum: Data Sufficiency
Topic: DS question
Replies: 8
Views: 3906

Thanks a million...thats what I wanted to ask! Pity GMAC doesn't tests Imaginary numbers!

Imaginary numbers will turn all equations around! Thanks for clarification!

by vinamra

Mon Jan 04, 2010 12:21 am
Forum: Data Sufficiency
Topic: Regarding Imaginary Numbers
Replies: 2
Views: 1319

Regarding Imaginary Numbers

This is from Princeton Review Drill Is X positive? i) xy=6 ii) x(y^2)=12 So, Options A and D are out since no use of that statement! Option B C E remains technically we can get answer from answer B, So option C and Option E are out! Here is my question: 1) Can we use imaginary Numbers ? (eg, √ -2 ...

by vinamra

Sun Jan 03, 2010 11:49 pm
Forum: Data Sufficiency
Topic: Regarding Imaginary Numbers
Replies: 2
Views: 1319

hey...awesome post/debrief....one of the best I have ever read! BTW its the craziness that drives one towards the goal. Though I am not for MBA (not now....but aiming for IE -School, Spain) even though I have 3 years exp....I am really trying hard for Msc Finance course from LSE (London School of Ec...

by vinamra

Wed Dec 30, 2009 10:16 am
Forum: I just Beat The GMAT!
Topic: GMAT 720 95%(Q49,V40)(people never fail - they just give up)
Replies: 384
Views: 400648

Thank you for your suggestions. Here is my plan: 1) Buy Princeton review and get brushed up with all my basic Math concepts till 7th January. 2) Give my 1st free GMAT test from Princeton review 8th January. 3) Complete analysis of score and further plan of action. 4) Buy OG 12th edition and Manhatta...

by vinamra

Tue Dec 29, 2009 6:36 pm
Forum: GMAT Strategy
Topic: Recommend books for starters!
Replies: 4
Views: 3137

OMG! Now I get it! That was so stupid of me!

BTW Thanks a Million Stuart Kovinsky for helping me out!

by vinamra

Tue Dec 29, 2009 9:22 am
Forum: Problem Solving
Topic: Number line with must be true question
Replies: 6
Views: 3827

Equally correct answers would have been: a > -1000000 a doesn't equal 53 a < 745.329 In fact all the alternate answers give by you are Correct, because it hasn't got any "=" sign.... the following options would be wrong: a=> -1000000 (because in no way a= -1000000) a<= 745.329 (because in...

by vinamra

Mon Dec 28, 2009 9:21 pm
Forum: Problem Solving
Topic: Number line with must be true question
Replies: 6
Views: 3827

This problem is from Princeton Review if a<=6 and a>=-5, then which of the following must be true for all values of a? a) a<=5 b) a>=-6 c) a<= -5 d) -6<=a<,5 e) none of the above My answer is E but solution is B I don't get why it's B. Since a>=-6 so A could be -6 which is false because a>=-5 thank...

by vinamra

Mon Dec 28, 2009 9:17 pm
Forum: Problem Solving
Topic: Number line with must be true question
Replies: 6
Views: 3827

Recommend books for starters!

Hi, After going through this forum, which is quite amazing, I still have some doubts unanswered. Following are the details which will help you to answer my questions: 1) GMAT attempt date: Around Mid-March, 2010 2) Target: 720+ 3) Excellent at Maths, though I need some brush up 4) Good at English, a...

by vinamra

Fri Dec 25, 2009 11:22 am
Forum: GMAT Strategy
Topic: Recommend books for starters!
Replies: 4
Views: 3137